GMAT Verbal Ques

40
The Central City Church building was recently damaged by a fire that occurred overnight. The insurance company will not pay on a claim if it is determined that a fire was started deliberately. Since nobody was in the Central City Church building at the time the fire swept through the structure, the insurance company will most certainly reimburse the church for the damage caused by the fire. Which of the following is an assumption that is required to reach the conclusion above?  A recent su rvey cond ucted in the New Homes magazine indicated that more than 70% of its readers are planning to purchase or build a new home over the next two years. Based on the results of the survey, the editor of the magazine concluded that over the next two years, the real estate industry is likely to experience rapid growth driven by the high market demand for new homes. Which of the following statements, if true, would be the most damaging to the conclusion of the magazine’s editor? There is only one major road, Freeway Z, that links County X and County Y. The border of the two counties is primarily defined by a mountain range, over which the construction of new roads is severely restricted by environmental laws. A cost-effective solution to the problem of traffic congestion on Freeway Z is to build a commuter train tunnel through the mountain range. The successful implementation of this plan would cost far less than expanding the existing freeway and would also reduce the number of cars clogging the roads in both counties. Which of the following, if true, could proponents of the plan above most appropriately cite as a piece of evidence for the soundness of their plan? Fire damages are generally the most expensive type of loss that insurance companies cover. There was no one in the building at the time the fire swept through the structure. No individual would fraudulently profit by setting a fire in the church building. The insurance company typically pays claims for s imilar situations in other non-residential structures, such as office buildings and restaurants. The fire was not deliberately started by someone who then left the building before the fire grew The survey included over 50,000 respondents representing all major age categories. The survey was mailed to all the subscribers of the magazine and included prepaid postage to facilitat e returning the questionnaire. Readers of the New Ho mes magazine are much more likely to be interested in buying or purchasing a home than the rest of the public. The number of the magazine's readers has more than doubled over the past 2 years. All survey respondents received one free issue of the magazine as a token of appreciation for filling out the survey.

Transcript of GMAT Verbal Ques

Page 1: GMAT Verbal Ques

7/30/2019 GMAT Verbal Ques

http://slidepdf.com/reader/full/gmat-verbal-ques 1/40

The Central City Church building was recently damaged by a fire that occurred overnight. The

insurance company will not pay on a claim if it is determined that a fire was started deliberately.

Since nobody was in the Central City Church building at the time the fire swept through the structure,

the insurance company will most certainly reimburse the church for the damage caused by the fire.

Which of the following is an assumption that is required to reach the conclusion above?

 A recent survey conducted in the New Homes magazine indicated that more than 70% of its readers

are planning to purchase or build a new home over the next two years. Based on the results of the

survey, the editor of the magazine concluded that over the next two years, the real estate industry is

likely to experience rapid growth driven by the high market demand for new homes.

Which of the following statements, if true, would be the most damaging to the conclusion of the

magazine’s editor? 

There is only one major road, Freeway Z, that links County X and County Y. The border of the two

counties is primarily defined by a mountain range, over which the construction of new roads is

severely restricted by environmental laws. A cost-effective solution to the problem of trafficcongestion on Freeway Z is to build a commuter train tunnel through the mountain range. The

successful implementation of this plan would cost far less than expanding the existing freeway and

would also reduce the number of cars clogging the roads in both counties.

Which of the following, if true, could proponents of the plan above most appropriately cite as a piece

of evidence for the soundness of their plan?

Fire damages are generally the most expensive type of loss that insurance companies cover.

There was no one in the building at the time the fire swept through the structure.

No individual would fraudulently profit by setting a fire in the church building.

The insurance company typically pays claims for similar situations in other non-residential structures,

such as office buildings and restaurants.

The fire was not deliberately started by someone who then left the building before the fire grew

The survey included over 50,000 respondents representing all major age categories.

The survey was mailed to all the subscribers of the magazine and included prepaid postage to facilitatereturning the questionnaire.

Readers of the New Homes magazine are much more likely to be interested in buying or purchasing a

home than the rest of the public.

The number of the magazine's readers has more than doubled over the past 2 years.

All survey respondents received one free issue of the magazine as a token of appreciation for filling outthe survey.

Page 2: GMAT Verbal Ques

7/30/2019 GMAT Verbal Ques

http://slidepdf.com/reader/full/gmat-verbal-ques 2/40

In a certain country, insurance companies pay only a fixed percentage – set by law and uniform

across all insurance companies – of the fees actually billed by healthcare providers. Patients without

insurance coverage, however, are legally responsible for the full amount of those fees. If the fixed

percentage set for insurance companies is changed, healthcare providers will adjust their fees so

that the insurance companies pay the same dollar amount that they would have paid before the

change.

The information given most strongly supports which of the following general claims about the country

described?

Sentence Correction 

Each Sentence Correction question presents a sentence, part of which or all of which is underlined.

Beneath the sentence you will find five ways of phrasing the underlined part.

 Answer choice (A) repeats the original; answer choices (B), (C), (D), and (E) are different.

If you think the original is best, choose answer choice (A); otherwise choose one of the other answer 

choices--whichever is best.

An effective commuter train tunnel between the counties would require major investment in mass

transit within both counties.

The majority of all vehicles on the nation’s freeways are traveling from one state to another. 

60% of the cars on Freeway Z are driven by people who live in County Y and work in County X.

Many new freeways are being built in areas that are presently served by commuter trains.

A large proportion of the vehicles on Freeway Z are commercial trucks carrying transcontinental

shipments.

If the percentage prescribed by law for insurance companies is decreased, private patients without

insurance coverage will pay more for healthcare services.

The dollar amount paid by insurance companies to healthcare providers will not change unless the

legally specified percentage is changed.

If the number of patients without insurance coverage grows beyond the number of patients covered by

insurance, healthcare providers will adjust their billing practices according to the needs of thosepatients.

If an economic recession causes the healthcare industry to suffer, the legally prescribed percentage

paid by insurance companies will be increased.

Acquiring insurance coverage, if possible, would be a sound economic decision for any patient.

Page 3: GMAT Verbal Ques

7/30/2019 GMAT Verbal Ques

http://slidepdf.com/reader/full/gmat-verbal-ques 3/40

The best answer choice is the one that conforms to the rules of standard written English and

produces the most effective sentence. This answer should be clear and exact, without ambiguity,

redundancy, or grammatical error.

Like in 2004, car sales to first-time buyers as often, if not more often than, to return customers

buoyed the economy this January.

Reading Comprehension 

Reading Comprehension questions test your ability to answer questions based on reading

passages. You will see several reading passages in the verbal section of the exam.

For each passage, you will be presented with a group of questions based on the content of that

passage. Each question will have 5 answer choices. Select the best answer to each question on the

basis of what is stated or implied in the particular passage.

Symptoms of Parkinson’s Disease, such as tremors, are thought to be

caused by low dopamine levels in the brain. Current treatments of Parkinson’s

disease are primarily reactionary, aiming to replenish dopamine levels after

dopamine-producing neurons in the brain have died. Without a more detailed

understanding of the behavior of dopamine-producing neurons, it has been

impossible to develop treatments that would prevent the destruction of these

neurons in Parkinson’s patients. 

Recent research provides insight into the inner workings of dopamine-

producing neurons, and may lead to a new drug treatment that would

proactively protect the neurons from decay. By examining the alpha-synuclein

protein in yeast cells, scientists have determined that toxic levels of the protein

have a detrimental effect on protein transfer within the cell. More specifically,

high levels of alpha-synuclein disrupt the flow of proteins from the endoplasmic

reticulum, the site of protein production in the cell, to the Golgi apparatus, the

component of the cell that modifies and sorts the proteins before sending them

Like in 2004, car sales to first-time buyers as often, if not more often than, to return customers buoyed

Like in 2004, first-time buyers bought cars as often, if not more often than, return customers and that

buoyed

As in 2004, car sales to first-time buyers as often as, if not more often than, to return customers and it

buoyed

As in 2004, first-time buyers bought cars as often as, if not more often than, return customers, buoying

As in 2004, car sales to first-time buyers as often, if not more often than, to return customers buoyed

Page 4: GMAT Verbal Ques

7/30/2019 GMAT Verbal Ques

http://slidepdf.com/reader/full/gmat-verbal-ques 4/40

to their final destinations within the cell. When the smooth transfer of proteins

from the endoplasmic reticulum to the Golgi apparatus is interrupted, the cell

dies.

With this in mind, researchers conducted a genetic screen in yeast cells

in order to identify any gene that works to reverse the toxic levels of alpha-

synuclein in the cell. Researchers discovered that such a gene does in fact exist,

and have located the genetic counterpart in mammalian nerve cells, or neurons.

This discovery has led to new hopes that drug therapy could potentially activate

this gene, thereby suppressing the toxicity of alpha-synuclein in dopamine-

producing neurons.

While drug therapy to suppress alpha-synuclein has been examined in

yeast, fruitflies, roundworms, and cultures of rat neurons, researchers are

hesitant to conclude that such therapies will prove successful on human

patients. Alpha-synuclein toxicity seems to be one cause for the death of 

dopamine-producing neurons in Parkinson’s patients, but other causes mayexist. Most scientists involved with Parkinson’s research do agree, however, that

such promising early results provide a basis for further testing.

The primary purpose of the passage is to 

Which of the following statements regarding a high level of alpha-synuclein is supported by thepassage? 

 According to the passage, which of the following represents the chronology of a typical protein life ina healthy yeast cell? 

compare and contrast current treatments for Parkinson’s Disease 

discuss new scientific findings and the implications of these findings for treating Parkinson’s Disease 

explain the role of proteins in dopamine-producing neurons

introduce new research that supports an already existing treatment method for Parkinson’s Disease 

question the scientific evidence used to support current treatments for Parkinson’s Disease 

It is not a cause for the death of neurons in Parkinson’s patients, though it has led to other research that

may prove fruitful.

Its effect on human neurons cannot be tested.

It is one possible cause for the death of neurons in Parkinson’s patients, but not the only one. 

Its detrimental effect on human neurons can be prevented through drug therapy.

It is the only possible cause for the death of neurons in Parkinson’s patients.

Protein is made in the endoplasmic reticulum, sent to the Golgi apparatus, processed and altered in the

Golgi apparatus, and then delivered to other parts of the cell.

Page 5: GMAT Verbal Ques

7/30/2019 GMAT Verbal Ques

http://slidepdf.com/reader/full/gmat-verbal-ques 5/40

The Bunsen burner, named after the German scientist who improved its design and efficiency, was

invented in 1885 not by Bunsen himself but by fellow scientist Michael Faraday.

Named for the capital of Belgium, Brussels sprouts, which at its fullest growth scarcely exceeds a

large walnut in size, are immature buds shaped like tiny cabbages.

Protein is created in the Golgi apparatus, modified and delivered to other parts of the cell, then

decomposed by alpha-synuclein.

Protein is produced in the endoplasmic reticulum, sent to the Golgi apparatus, and then decomposed by

alpha-synuclein.

Protein is produced in the Golgi apparatus, modified by the Golgi apparatus, distributed to the neuron,

and then sent to the endoplasmic reticulum.

Protein is produced by alpha-synuclein, transferred to the endoplasmic reticulum, sent to the Golgi

apparatus for modification, and then distributed to the rest of the cell.

named after the German scientist who improved its design and efficiency, was invented in 1885 not by

Bunsen himself but by fellow scientist Michael Faraday

which was named for the German scientist who improved its design and efficiency, was not invented

in 1885 by Bunsen himself but, rather, by Michael Faraday, his fellow scientist

which is named for the German scientist improving its design and efficiency, was invented not by

Bunsen himself but, rather, by Michael Faraday, a fellow scientist in 1885

named for the German scientist improving its design and efficiency, was not invented by Bunsen

himself but by Michael Faraday, a fellow scientist, in 1885

naming after the German scientist who had improved its design and efficiency, was invented not byBunsen himself, but, rather, by fellow scientist Michael Faraday in 1885

its fullest growth scarcely exceeds a large walnut in

its fullest growth scarcely exceed a large walnut in

their fullest growth scarcely exceeds a large walnut in

their fullest growth scarcely exceed a walnut's large

their fullest growth scarcely exceed a large walnut in

Ecologist: The incidence of alligator attacks on humans in the vicinity of the Blue Lagoon has

increased in recent years. Relocating at least half of the Blue Lagoon’s alligator population

would help decrease the incidence of alligator attacks on humans.

The ecologist’s claim relies on which of the following assumptions? 

Alligators prefer humans to other food sources.

Page 6: GMAT Verbal Ques

7/30/2019 GMAT Verbal Ques

http://slidepdf.com/reader/full/gmat-verbal-ques 6/40

 

There is a correlation between the size of the alligator population and the incidence of 

alligator attacks on humans.

In recent years, there has been no significant change in the size of the alligator population in

the Blue Lagoon.

Not all alligator attacks on humans are reported to authorities.

Relocating half of the lagoon’s alligator population would not be prohibitively expensive. 

Martin Luther King Jr. won the Nobel Peace Prize, the Presidential Medal of Freedom, and

was the most famous leader of the American civil rights movement.

Martin Luther King Jr. won the Nobel Peace Prize, the Presidential Medal of Freedom, and

was the most famous leader of the American civil rights movement.

Martin Luther King Jr., the most famous leader of the American civil rights movement, won

the Nobel Peace Prize and the Presidential Medal of Freedom.

Martin Luther King Jr., the most famous leader of the American civil rights movement, wonthe Nobel Peace Prize and he won the Presidential Medal of Freedom.

Martin Luther King Jr. was the most famous leader of the American civil rights movement,the winner of the Nobel Peace Prize, and he won the Presidential Medal of Freedom.

Martin Luther King Jr., the most famous leader of the American civil rights movement, won

the Nobel Peace Prize as well as the Presidential Medal of Freedom, too.

 Although amyotrophic lateral sclerosis, or ALS, typically causes death within 3 to 5 years of 

the onset of symptoms, on average, the famous physicist Stephen Hawking has defied the

odds, in contrast, by living more than 40 years after his initial diagnosis.

typically causes death within 3 to 5 years of the onset of symptoms, on average, the famous

physicist Stephen Hawking has defied the odds, in contrast,

causes death within 3 to 5 years of the onset of symptoms, on average, the famous physicist

Stephen Hawking has defied the odds, in contrast,

typically causes death within 3 to 5 years of the onset of symptoms, the famous physicistStephen Hawking has defied the odds

causes death within 3 to 5 years of the average onset of symptoms, the famous physicist

Stephen Hawking has defied the odds, by contrast,

typically causes death within 3 to 5 years of the average onset of symptoms, the famous

physicist Stephen Hawking has defied the odds

The golden toad of Costa Rica, whose beauty and rarity inspired anunusual degree of human interest from a public generally unconcernedabout amphibians, may nevertheless have been driven to extinction byhuman activity. In the United States, a public relations campaign raisedmoney to protect the toad’s habitat in Costa Rica, establishing theMonteverde Cloud Forest Preserve in 1972. However, setting aside habitatwas not enough to save the species. The toad's demise in the late 1980s

Page 7: GMAT Verbal Ques

7/30/2019 GMAT Verbal Ques

http://slidepdf.com/reader/full/gmat-verbal-ques 7/40

was a harbinger of further species extinction in Costa Rica. Since thattime, another twenty of the fifty species of frogs and toads known to once

inhabit a 30 square kilometer area near Monteverde have disappeared.Between one third and one half of the world’s amphibian species—

including frogs, toads, and salamanders—have declined or disappeared.

Scientists hypothesize that the more subtle effects of human activities onthe world's ecosystems, such as the accretion of pollutants, the decreasein atmospheric ozone, and changing weather patterns due to globalwarming, are beginning to take their toll. Perhaps amphibians - whosepermeable skin makes them unusually sensitive to environmental changes- are the biological harbingers of the natural world, giving humans early

notification of the deterioration, if not destruction, of our ecosystem.In the second paragraph, the author notes that amphibians have permeable skin in order to 

argue that the golden toad is not the only endangered amphibian

explain why amphibians might be affected earlier than other animals by an adverse

environmentillustrate that preserving the golden toad’s habitat was not enough to save the species 

demonstrate that the accretion of pollutants poses a greater threat to amphibian species than

do changes to the atmosphere or climate

suggest that threats to amphibians need not be a cause for human concern

Which of the following best describes the overall organization of the passage?  

The first paragraph speculates on the causes of a global phenomenon; the second paragraph

describes one specific example of this phenomenon.

The first paragraph explains one environmental consequence of human activity; the second

paragraph discusses solutions which might mitigate further environmental damage.

The first paragraph details one instance of an ecological change; the second paragraph

discusses the import and global extent of this change.

The first paragraph discusses efforts to protect the habitat of a particular species; the second

paragraph advocates for additional habitat protection.

The first paragraph describes the unexplained worldwide disappearance of toads; the secondparagraph adds that salamanders and frogs are also disappearing.

 According to the passage, all of the following are true EXCEPT: 

Humans are at least partially responsible for changing weather patterns.

Toads, like frogs, have permeable skin.

A species may become extinct even if its habitat is preserved.

Costa Rica’s Monteverde Cloud Forest Preserve was not paid for solely by the Costa Rican

government.

More frog and toad species than salamander species have disappeared in Costa Rica sincethe late 1980s.

Page 8: GMAT Verbal Ques

7/30/2019 GMAT Verbal Ques

http://slidepdf.com/reader/full/gmat-verbal-ques 8/40

The recent global boom in the market price for scrap steel and aluminum leads to a sudden rise in

the theft of everyday metal objects like manhole covers, guard rails, and empty beer kegs.

Discouraged by new data that show increases in toxic emissions from domestic factories, searches

for alternative investment opportunities are being conducted by shareholders of the nation’s leadingmanufacturing companies.

The capital gains tax, levied against profits earned on investments, have been being the subject of 

recent political debate.

leads to a sudden rise in the theft of everyday metal objects like manhole covers, guard rails, andempty beer kegs

have led to a sudden rise in the theft of everyday objects such as manhole covers, guard rails, and

empty beer kegs

has led to a sudden increase in the number of thieves for everyday metal objects like manhole covers,

guard rails, and empty beer kegs

has led to a sudden rise in the theft of everyday metal objects such as manhole covers, guard rails, and

empty beer kegs

is leading the suddenly rising theft of everyday metal objects: manhole covers, guard rails, and empty

beer kegs

searches for alternative investment opportunities are being conducted by shareholders of the nation’s

leading manufacturing companies

searches are being conducted by shareholders of the nation’s leading manufacturing companies who

are looking for alternative investment opportunities

shareholders of the nation’s leading manufacturing companies had begun searching for investment

opportunities outside of the manufacturing industry

the nation’s leading manufacturing companies are searching for alternative investment opportunities

for its shareholders

shareholders of the nation’s leading manufacturing companies are searching for alternative investmentopportunities

have been being

have been

were

has been being

has been

Items that seem unremarkable today might once have altered thecourse of history. For centuries, the nutmeg tree grew only in the Banda

Islands, a small chain in the southwest Pacific. Locals harvested thearomatic nuts of the tree and sold them to traders. Eventually, a spice

made from these nuts became a luxury item in the European market, via

Page 9: GMAT Verbal Ques

7/30/2019 GMAT Verbal Ques

http://slidepdf.com/reader/full/gmat-verbal-ques 9/40

Which of the following is mentioned in the passage as a reason for the initial interest of theNetherlands in the Banda Islands? 

Venetian merchants. Seeking a monopoly over this valuable spice, theDutch attacked the Banda Islands, subjugating the native people in a

mostly successful attempt to control the trade.However, one island in the Banda chain remained in the hands of 

the British and was the object of much conflict between the Netherlands

and England. After many battles, the British offered to cede control of theisland in exchange for New Amsterdam, a Dutch outpost on the east coastof North America. Inveterate traders, the Dutch were more interested inthe spice trade than in the small outpost of New Amsterdam. In 1667, theTreaty of Breda gave the Dutch complete control of the Banda Islands,and thus of the nutmeg trade, and gave the British New Amsterdam,

which they promptly renamed New York. Today, nutmeg trees can befound in many countries and no one company or country has a monopoly

on the trade.

Th

eauthor mentionsthatnutmegbec

ame a luxury item in the European market in order to  

discuss the role of the Dutch in the history of the trade of luxury items

explain why the Dutch wanted to control the market for nutmeg

establish that the Dutch obtained a monopoly on the importation of nutmeg into Europe

demonstrate the importance of spices in Europe

explain why the British wanted to control the market for nutmeg

In the passage, the author is primarily interested in 

tracing the history of a major city

discussing the role of commonplace items in world development

offering a specific example to support a general claim

arguing for continued research into political history

presenting an innovative view of a commonplace item

Increased economic competition with Britain

Subjugation of the native people

An opportunity to expand to a new geographical market

A failure to cultivate nutmeg in other locationsn

Page 10: GMAT Verbal Ques

7/30/2019 GMAT Verbal Ques

http://slidepdf.com/reader/full/gmat-verbal-ques 10/40

 Which of the following best completes the passage below?

In an exit poll of voters who had recently voted for governor, four-fifths said they would vote for a

gubernatorial candidate who uses a wheelchair. However, the poll may have overestimated theproportion of voters who would vote for such a candidate, because ______________.

 At any given time, approximately fifteen percent of all homes in Florida are on the market. In Texas,

however, only seven percent of all homes are on the market at any given time. Therefore, one will

have a wider selection of homes to choose from if one looks for a home in Florida rather than in

Texas.

Which of the following, if true, would most seriously strengthen the argument above?

The desire to restrict access to a commodity

Some voters who would not vote for a candidate who uses a wheelchair might have claimed to the

pollster that they would.

Some voters who would vote for a candidate who uses a wheelchair may not have accurately answered

the pollster’s questions.

Some voters who said they would not vote for a candidate who uses a wheelchair also would not votefor candidates who express certain types of beliefs.

Some people who are not registered voters also would probably not vote for a candidate who uses awheelchair.

Some voters who claim they would not vote for a candidate who uses a wheelchair might vote for a

candidate who uses a wheelchair if that candidate shared their beliefs.

Homes in Florida tend to be less expensive than those in Texas.

Mortgages are easier to obtain for homes in Florida than for homes in Texas.

The construction industry in Texas has reported significant growth over the past year.

The cost of constructing new homes in Texas is higher than in Florida.

The total number of homes in Florida is three times greater than the total number in Texas.

Kleine-Levin syndrome, a rare neurological disorder primarily affecting adolescents, is

characterized by cyclical episodes of extreme lethargy, heightened irritability, and appearing

confused.

appearing confused

they appear confused

may be confused

Page 11: GMAT Verbal Ques

7/30/2019 GMAT Verbal Ques

http://slidepdf.com/reader/full/gmat-verbal-ques 11/40

 Albinism is a rare genetic condition that inhibits the production of melanin, or pigmentation, in theskin and hair. People born with albinism are unusually susceptible to sunburn, melanoma, and a

range of other health issues that are generally connected to excessive exposure to the sun.

The statements above, if true, provide the most support for which of the following conclusions?

apparent confusion

had been confused

Studies show thatrepeated exposure toan allergen cantrigger anaphylaxis.This condition, inwhich an allergicperson becomesoverly sensitized tothe allergen inquestion, can provokea life-threatening

reaction via evenminimal exposure.

Because minor earthquakes can weaken the existing infrastructure in an area, a

series of minor earthquakes sometimes causes much more extensive damage thanmight otherwise be expected.

Peanut allergy sufferers do not need to ingest peanut products in order toexperience an allergic episode; a reaction can also be triggered by touching peanuts

or peanut oils.

Scientists theorize that global warming, a gradual increase in the average

worldwide temperature, is the primary cause of the increasing occurrence of extreme weather patterns in the last two decades.

People born with albinism develop other biological protections against melanoma and other sun-related

health issues.

Humans with a high production of melanin can easily ignore health issues related to exposure to the

sun.

When a non-albino person gets sunburn, the amount of melanin produced by that person decreases.

In humans, melanin plays a role in protecting the skin from developing sunburn and other sun-related

ailments.

It is not possible for a person born with albinism to adopt other artificial protective measures against

excessive exposure to the sun.

Page 12: GMAT Verbal Ques

7/30/2019 GMAT Verbal Ques

http://slidepdf.com/reader/full/gmat-verbal-ques 12/40

Regular maintenance such as inflating tires, replacing air filters, and tuning the engine can not only

extend the life of a vehicle, but also can boost fuel efficiency.

Which of the following is the best analogy for the process described above?

not

onlyextend

the lifeof a

vehicle,

butalso

can

not

only

extendthe life

of a

vehicle,

but

also

extend

the life

of a

vehicle,

butalso

can

extend

notonly

the life

of avehicle,

but can

not

onlyextend

the lifeof a

vehicle,

andalso

can

People with unhealthy diets are much more susceptible to conditions such as

diabetes and high blood pressure than is the general population.

Page 13: GMAT Verbal Ques

7/30/2019 GMAT Verbal Ques

http://slidepdf.com/reader/full/gmat-verbal-ques 13/40

Parents are concerned not only with finding good schools for their children, but they have to know

what makes one better than another.

but they have to know what makes one better than another

but they must decide which school is better than the other

and knowing what school is best of them all

and they have to know what makes one school better than another

but also with knowing what makes one school better than another

The movement for women’s rights traces its origin to the first half of 

the nineteenth century. The Seneca Falls Convention, held in Seneca Falls,New York in July of 1848, is commonly regarded as the beginning of the

women's rights movement in the United States. This conference waspreceded by a series of ground-breaking events that made possible thisseminal milestone in the history of American women.

The idea for the convention emerged during the 1840 World Anti-Slavery Convention in London, a conference that precluded its femaledelegates from participation in discussions. Lucretia Mott, a famouswomen’s rights activist, wrote in her diary that calling the 1840convention a “world” convention "was a mere poetical license." She hadaccompanied her husband to London but had to sit behind a partition withother women activists, including Elizabeth Cady Stanton, who laterbecame one of the main forces behind the Seneca Falls Convention.

During the early 1840s, Elizabeth Cady Stanton composed theDeclaration of Sentiments, a document modeled after the Declaration of 

Independence, declaring the rights of women. At the time of itscomposition, the Declaration of Sentiments was so bold that when

Elizabeth Stanton showed the draft to her husband, he stated that if sheread it at the Seneca Falls Convention, he would have to leave town. TheDeclaration contained several new resolutions. It proclaimed that all menand women are born equal and stated that no man could withhold awoman's rights, take her property, or preclude her from the right to vote.

This Declaration also became the foundation for the Seneca Falls

Convention.On July 19-20, 1848, the Seneca Falls Convention brought together240 delegates between ages 22 and 60, including forty men, who spent

the two days at the conference debating, refining and voting on theDeclaration of Sentiments. Most of the declaration’s resolutions received

unanimous support and were officially endorsed. Later in 1848, theSeneca Falls convention was followed by an even larger meeting in

Rochester, New York. Thereafter, national women's conventions were held

Page 14: GMAT Verbal Ques

7/30/2019 GMAT Verbal Ques

http://slidepdf.com/reader/full/gmat-verbal-ques 14/40

The author of the passage mentions Stanton's husband's intention to leave town most likely in order to emphasize which of the following? 

 According to the passage, the articles of the Declaration of Sentiments precluded each of thefollowing, EXCEPT: 

annually, contributing to the growing momentum in the movement forwomen's rights.Which of the following best describes the main purpose of the passage above? 

To compare the origin of the women’s rights movement with the current-day situation.

To support further expansion of women’s rights in the United States.

To criticize the nineteenth-century restrictions on women’s rights. 

To explain the reasons for the opposition to the Seneca Falls Convention.

To discuss the origin of the women’s rights movement. 

The personal relationship between Stanton and her husband was rapidly deteriorating.

Stanton's husband needed to attend another convention that was held out of town.

Stanton's husband was prone to spontaneous and emotional decisions.

The Declaration of Sentiments contained bold proclamations for its time.

The abusive language in the Declaration of Sentiments could reflect negatively on the reputation of 

Stanton’s husband. 

Restraining women from participating in military expeditions.

Denying women the right to vote.

Taking women’s property. 

Claiming that men and women are not born equal.

Withholding women’s rights. 

Congress has passed a law that scales back the tax credits offered to purchasers of hybrid

cars. Because of this, the number of hybrid cars sold each year, which has approximately

doubled in each of the last five years, is likely to flatten in the next few years.

Which of the following statements, if true, most weakens the conclusion that the number of 

hybrid cars sold annually is likely to flatten in the next few years?

Most people buy hybrid cars for reasons other than to save money in the long run.

Page 15: GMAT Verbal Ques

7/30/2019 GMAT Verbal Ques

http://slidepdf.com/reader/full/gmat-verbal-ques 15/40

Small community hospitals in poor urban areas almost always operate at a loss due to an

unfortunate cycle of factors. High revenue specialists, such as surgeons, flock to hospitals that are

more prestigious and can afford to pay higher salaries. Aware of this, local residents patronize the

more affluent nearby hospitals when they need specialty care or forgo care entirely, while only

utilizing the local hospital for low-margin routine care. Further, a significant majority of the

community uses government health plans, which reimburse poorly for routine care, or lacks

insurance entirely and cannot pay. The local hospital then loses money and cannot afford to hire

specialists to conduct the higher-margin specialty care.

Recently published studies have shown that hybrid cars cost significantly more to repair and

maintain than do comparable traditional cars.

Gasoline prices are projected to fall over the next few years.

Hybrid vehicles have a reputation for having less power than comparable non-hybrid

vehicles.

Manufacturers have significantly increased their budgets for advertisements touting the fuel

efficiency of hybrid cars

Hoping to alleviate some of the financial burdens of a growing population, property taxes last

year were raised by an eleven percent increase by the county government.

property taxes last year were raised by an eleven percent increase by the county government

property taxes were raised by eleven percent last year by the county government

the county government raised property taxes by an eleven percent increase last year

the county government last year raised by eleven percent property taxes

the county government raised property taxes by eleven percent last year

The increase in unemployment rates, coupled with significantly increased retail prices as well

as energy costs, are forcing many homeowners to look into alternative sources of fuel in

order to save money on winter heating.

unemployment rates, coupled with significantly increased retail prices as well as energy

costs, arerates of unemployment, coupled with significant increases in retail prices and energy costs,

have been

unemployment rates, coupled with significant increases in both retail prices and energy

costs, is

unemployment rates, coupled with significantly increased retail prices as well as energy

costs, is

rates of unemployment, coupled with significant increases in both retail as well as energycosts, had been

Page 16: GMAT Verbal Ques

7/30/2019 GMAT Verbal Ques

http://slidepdf.com/reader/full/gmat-verbal-ques 16/40

 

Which of the following, if it could be accomplished, would best help small community hospitals to

break the pattern described above?

This year, the number of applications at College X increased by nearly 5%, while the size of the

entering class and the structure of the admissions process remained the same. However, the

admissions director of College X claims that this year a greater proportion of its applicants

received offers of admission from the college than in years past.

Which of the following statements best reconciles the claim of the admissions director with o ther 

evidence presented in the argument?

The biathlon is one of the world’s most demanding sports, it combines the endurance of cross

country skiing with the precision of expert marksmanship.

Negotiate higher reimbursement rates for specialty care with both government health plans and privateinsurers.

Advertise the hospital's specialty care services in the local community as well as nearby communities

to attract more business.

Partner with a nearby affluent hospital to contract its specialists on a part-time, as-needed basis, which

is more affordable than hiring these specialists full time.

Launch a community outreach campaign to educate the public about the low reimbursement rates of 

government health plans compared with the high rates paid by private insurers.

Negotiate with private insurers for higher reimbursement rates for routine care.

This year College X moved up in all major rankings of undergraduate programs.

This year, the number of high school graduates who applied to colleges increased by 5%.

This year, nearly twice as many accepted applicants at College X decided to attend other colleges.

The admissions process at College X is substantially more competitive, as compared to other colleges

of similar academic profile.

College X is located in a major metropolitan area with a high concentration of potential college

applicants.

sports, it combines

sports; that combines

sports; it combines

sports, where they combine

sports in which they combine

Page 17: GMAT Verbal Ques

7/30/2019 GMAT Verbal Ques

http://slidepdf.com/reader/full/gmat-verbal-ques 17/40

If the fire in the church building was started deliberately, the insurance company will not pay for the

damages. In order to conclude that the insurance company will pay, one must assume that the fire

was not set intentionally.

(A) The fact that fire damages tend to be more expensive for insurance companies than other losses

does explain why the insurance company may be reluctant to pay the claim. However, this fact is

The director of programming at NNN, the National News Network, proclaimed that, despite

charges to the contrary, the Network does not have a conservative bias. The director 

acknowledged that, when a liberal news personality was recently added to the Network, an

additional conservative news program was also established. However, the director argued

that far from demonstrating any bias, these actions reflect a commitment to presenting abalanced perspective in interpreting current events.

Which of the following pieces of information would be most useful in evaluating the validity of 

the programming director’s argument? 

The Network has established an additional liberal news program whenever an additional

conservative news personality has been added.

The Network airs a program focused largely on the opinions of its viewers on contemporary

issues.

The newly added liberal news personality came from a rival network with an acknowledgedliberal bias.

The newly added liberal news personality has indicated that he has not felt any editorialpressure in his new environment.

The viewer ratings of the Network have been lower this past year than in previous years.

The portion of the interest earned on the state university's endowment that is set aside to

fund merit scholarships for outstanding high school seniors is more numerous than that set

aside to fund the university's high-powered athletic program.

is more numerous than

are more numerous than

is greater than

is greater

are greater than

Page 18: GMAT Verbal Ques

7/30/2019 GMAT Verbal Ques

http://slidepdf.com/reader/full/gmat-verbal-ques 18/40

Page 19: GMAT Verbal Ques

7/30/2019 GMAT Verbal Ques

http://slidepdf.com/reader/full/gmat-verbal-ques 19/40

in real estate, thus supporting rather than weakening the argument of the author.

(E) While receiving modest compensation for filling out the survey could potentially increase the

response rate, this factor is unlikely to affect the results of the survey and is therefore outside the

scope of the argument.

Since construction of new roads between County X and County Y is severely restricted, this

argument claims that building a commuter train between the counties is a more cost-effective way to

reduce congestion on Freeway Z than expanding the existing freeway or building a new one. This

plan will work only if people currently driving on the freeway are likely to use the new commuter train.

If the congestion is caused largely by people driving between the two counties, then some of these

people would likely choose to use the train instead, decreasing traffic congestion on the freeway.

(A) The cost associated with funding necessary mass transit in both counties, in addition to the

expense of the tunnel itself, is an argument against, not for, the proposed commuter train tunnel.

(B) This plan addresses a very specific situation: traffic between two counties on a certain freeway.

Information about the nation’s freeways in general is not relevant to this argument. 

(C) CORRECT. This statement shows that a majority of the cars on Freeway Z are driven by

commuters. Commuters are more likely than other groups to use the proposed train tunnel; if so,

fewer cars would travel on the freeway, and traffic congestion would decrease.

(D) This plan addresses a very specific situation: traffic between two counties on a certain freeway.

New freeways that are being built elsewhere are not relevant to this argument. Even if this

information were relevant to this situation, it would be an argument against the proposed commuter 

train tunnel, indicating that new freeway construction is necessary despite the existence of 

commuter trains.

(E) The information that Freeway Z is congested largely because of transcontinental shipments in

commercial trucks, rather than commuters between the two counties, is an argument against the

proposed commuter train tunnel. A large proportion of the vehicles on Freeway Z are commercial

trucks, which will still travel that route even if the commuter train tunnel is built, so traffic congestion

will be largely unaffected.

(1) Identify the Question Type  

 At first this may seem to be a Strengthen question, because of the phrase ―most strongly

supports.‖ However, we are asked which answer choice is supported by the information in

the passage, so this is an Inference question.

(2) Deconstruct the Argument  

Under the law, insurance companies only have to pay a fixed percentage of healthcare bills,

but private patients must pay the full amount. In addition, if the percentage is changed,

healthcare providers will adjust the amount of the bill, so that the dollar amount does not

change. If the legal percentage is decreased, healthcare providers must increase the billing

level in order to offset the change. Alternatively, if the legal percentage is increased,

Page 20: GMAT Verbal Ques

7/30/2019 GMAT Verbal Ques

http://slidepdf.com/reader/full/gmat-verbal-ques 20/40

healthcare providers would have to decrease the billing level in order to offset the change.

(3) State the Goal  

On inference questions, we’re looking for the answer that must be true according to the

information given in the passage. The most tempting wrong answers are often items that

might be true but don’t have to be true.

(4) Work from Wrong to Right  

(A) CORRECT. The passage states that, if the percentage is changed, healthcare providers

will adjust the amount of the bill so that the dollar amount does not change. In particular, if the

legal percentage is decreased, healthcare providers must bill increased amounts to

accomplish this objective. Since private patients are responsible for the full amount billed,

they will be responsible for the new, higher amount.

(B) There is nothing in the passage to suggest that the amount paid by insurance companies

could not change for reasons unrelated to the law. For example, nothing in the passageprevents the healthcare providers from changing their prices, and a change in prices without

a change in the percentage paid would result in a change in the dollar amount paid by the

insurance companies.

(C) This statement might be true, but there is nothing in the passage to indicate that it must

be true. The passage describes what healthcare providers will do only in the event of a

change in the law; it does not describe what they will do in other situations, such as the one

described in this answer choice.

(D) The passage provides no information about what determines the legally set percentage.

(E) This statement may be true for some patients but does not have to be true for all, e.g., for 

those who use healthcare services very infrequently. For such patients – even if they pay

greater bills when they do use healthcare services – the difference may not make up for the

cost of insurance premiums, especially if those premiums are high.

(2) The original makes a comparison between car sales in 2004 and this January. However, the

comparison is of prepositional phrases, which must be compared using "as," not "like," which

is used to compare nouns. ("Like" would be correctly used to compare one year to another,

for example, "Like 2004, 2005 was a good year.") Also, this sentence has an idiomatic

error. The idiom ―as often as‖ must be written out and cannot be contracted to ―as

often.‖ Finally, the phrasing "car sales to first-time buyers as often as to return customers" is

awkward and should be recast.

(A) This choice is incorrect as it repeats the original sentence.

(3) (B) This choice incorrectly uses the comparison term "Like." Also, the idiom ―as often‖ is

incorrect; it should be ―as often as.‖ 

Page 21: GMAT Verbal Ques

7/30/2019 GMAT Verbal Ques

http://slidepdf.com/reader/full/gmat-verbal-ques 21/40

(4) (C) This sentence uses the correct idiom, "as often as." However, the pronoun ―it‖ does not

have an antecedent, as ―sales‖ (as well as "buyers") is plural. Finally, "and it" weakens the

syntax and meaning of the first part of the sentence.

(5) (D) CORRECT. This choice clearly compares sales in the two years. The idiom ―as often as‖

is correctly written and is placed in a comparison of actions (i.e., "first-time buyers bought

cars") rather than in a comparison of prepositional phrases, which is more awkward.

(6) (E) The idiom ―as often‖ is incorrect; it should be ―as often as.‖ 

(7)  This is a general question that can be answered by considering the general structure of the

passage.

(A) This passage does not compare and contrast current treatments. The first paragraph

does mention current treatments, but these treatments are grouped together as treatments

that all work to replenish dopamine levels in the brain.

(B) CORRECT. The topic sentence of the second paragraph discusses new scientific

findings and their implications: ―recent research provides insight into the inner workings of 

dopamine-producing neurons, and may lead to a new drug treatment that would proactively

protect the neurons from decay.‖ 

(C) While the protein alpha-synuclein is discussed in the second paragraph, the role of this

protein is not the main focus of the passage.

(D) The new research method does not support an ―already existing‖ treatment method, but

rather a new, potential treatment method.

(E) The scientific evidence discussed in this passage supports a new treatment method; it

does not call into question currently existing treatment methods. (8)  In the fourth paragraph, it is stated that ―alpha-synuclein toxicity seems to be one cause for 

the death of dopamine-producing neurons in Parkinson’s patients, but other causes may

exist.‖ 

(A) This statement contradicts the information given in the fourth paragraph. The fourth

paragraph states that alpha-synuclein toxicity, or high levels of alpha-synuclein, seems to be

one cause for the death of neurons.

(B) The fourth paragraph states ―that such promising early results provide a basis for further 

testing,‖ implying that further testing on the cause of neuron death, and the related drug

treatment, can and will be conducted.

(C) CORRECT. This statement agrees with the information given in paragraph four: ―alpha-

synuclein toxicity seems to be one cause for the death of dopamine-producing neurons in

Page 22: GMAT Verbal Ques

7/30/2019 GMAT Verbal Ques

http://slidepdf.com/reader/full/gmat-verbal-ques 22/40

Parkinson’s patients, but other causes may exist.‖ 

(D) While paragraph four states that drug therapy to prevent alpha-synuclein toxicity has

been tested in yeast, fruitflies, roundworms, and cultures of rat neurons, there is no evidence

to suggest that drug therapy will prove effective in human neurons.

(E) This statement contradicts the information given in the fourth paragraph. The fourth

paragraph states that alpha-synuclein toxicity, or high levels of alpha-synuclein, seems to be

one cause for the death of neurons, but not the only one. (9)  The second paragraph states that ―high levels of alpha-synuclein disrupt the flow of proteins

from the endoplasmic reticulum, the site of protein production in the cell, to the Golgi

apparatus, the component of the cell that modifies and sorts the proteins before sending

them to their final destinations within the cell.‖ This implies that proteins in healthy cells are

produced in the endoplasmic reticulum, sent to the Golgi apparatus where they are modified,

and then shipped to the rest of the cell.

(A) CORRECT. This chronology is supported by the information given in the secondparagraph.

(B) Proteins are created in the endoplasmic reticulum, not the Golgi apparatus. Further, there

is no evidence to show that proteins in healthy cells are decomposed by alpha-synuclein.

(C) There is no evidence to show that proteins in healthy cells are decomposed by alpha-

synuclein.

(D) Proteins are created in the endoplasmic reticulum, not the Golgi apparatus.

(E) Proteins are created in the endoplasmic reticulum, not by alpha-synuclein. 

(10) The original sentence is correct. The contrast set off by the parallel structures "not … but"

should only contrast Bunsen and Faraday; the reference to the year 1885 should not be

included in that parallel structure, since the year in which the burner was invented is not part

of the indicated contrast. The modifier "in 1885" should be attached as directly as possible to

the action "was invented".

The idioms "named for" and "named after" are both correct, so there can be no eliminations

based on those idioms.

The past tense "improved" is the most logical tense for that verb, since the action in question

is a simple historical action.

(A) The original sentence is correct as written.

(B) The past tense "was named" is inappropriate because it suggests that the burner is no

longer named after Bunsen. In addition, "not invented in 1885" wrongly suggests that the

burner was invented in a different year. Finally, the constructions following "not" and "but"

are not parallel.

(C) The present participle "improving" is inappropriate. This type of participle adopts the

Page 23: GMAT Verbal Ques

7/30/2019 GMAT Verbal Ques

http://slidepdf.com/reader/full/gmat-verbal-ques 23/40

tense of the clause to which it's attached; since that clause ("is named") is explicitly in the

present tense, "improving" suggests that Bunsen is still making those improvements in the

present day. In addition, the modifier "a fellow scientist in 1885" wrongly suggests that

Faraday was only a fellow scientist during that one year, i.e., that Faraday wasn't a fellow

scientist in other years.

(D) The present participle "improving" is inappropriate. This type of participle adopts the

tense of the clause to which it's attached; since that clause ("is named") is explicitly in the

present tense, "improving" suggests that Bunsen is still making those improvements in the

present day. In addition, the placement of "in 1885" within the "not … but" structure wrongly

suggests that the year 1885 is being contrasted with another year.

(E) The active participle "naming" inappropriately suggests that the burner itself is naming

something. Additionally, the past perfect "had improved" is inappropriate, since there is no

subsequent action preceded in a relevant manner by that improvement. Finally, the

placement of "in 1885" within the "not … but" structure wrongly suggests that the year 1885

is being contrasted with another year. 

(11) The modifier "which at its fullest growth scarcely exceeds . . ." contains two words--

the singular pronoun "its" and the singular verb "exceeds"--that suggest that the subject of 

the modifier is singular. Both of these words are incorrect, since the subject of this modifier 

is actually the plural noun "Brussels sprouts."

(12) (A) This choice is incorrect as it repeats the original sentence.

(13) (B) This choice incorrectly uses the singular pronoun "its" to refer to the plural

noun "Brussels sprouts."

(14) (C) In this choice, the singular verb "exceeds" does not agree with the plural pronoun

"their" (which refers to the plural subject "Brussels sprouts").

(15) (D) The plural pronoun "their" and the plural verb "exceed" correctly refer to the plural

noun "Brussels sprouts." However, the use of the phrase "scarcely exceed a walnut's large

size" distorts the meaning of the original phrase "scarcely exceeds a large walnut in size." In

the original, it is clear that the comparison is between the size of a Brussels Sprout and the

size of a large walnut. In this choice, the comparison is changed to one between the size of a

Brussels sprout and the size of any walnut, all of which, according to this choice, are large.

(16) (E) CORRECT. This choice correctly uses the plural pronoun "their" and the plural

verb "exceed" to refer to the plural noun "Brussels sprouts."

(17) The ecologist concludes that relocating at least half of the alligator population will help curb

attacks on humans. This conclusion hinges on the assumption that the size of the alligator 

Page 24: GMAT Verbal Ques

7/30/2019 GMAT Verbal Ques

http://slidepdf.com/reader/full/gmat-verbal-ques 24/40

population is actually related to the incidence of attacks on humans. The correct answer will

state this assumption.

(A) The alligators’ preference has no bearing on whether the number of alligators in the

lagoon actually affects the frequency of attacks on humans. Therefore, the conclusion is not

dependent on this assumption.

(B) CORRECT. This choice explicitly states that there is a relationship between the size of 

the alligator population and the frequency of attacks on humans.

(C) The conclusion is not dependent on this assumption. In fact, this assumption would most

likely lead to a different conclusion altogether. If the size of the alligator population has

remained constant while the incidence of attacks has increased, it would be logical to

conclude that the size of the population does not affect the incidence of attacks.

(D) This assumption, true or not true, does not address the relationship between the alligator 

population and the incidence of attacks. Therefore, the conclusion is not dependent on thisstatement.

(E) The expense associated with relocating the alligators has no bearing on the relationship

between the alligator population and the incidence of attacks. Therefore, the conclusion is

not dependent on this statement. (18) The items in a list require parallelism. In this sentence, "Martin Luther King Jr. won"

applies to each item in the list, which works for the first two items but does not work for the

third, "was the most famous leader..."

(19) (A) This answer choice is incorrect as it repeats the original sentence.

(20) (B) CORRECT. "The most famous leader of the American civil rights movement" is

no longer placed in a list with the two prizes won; instead, it is correctly used as a clause

modifying "Martin Luther King Jr." The two prizes ("Nobel Peace Prize" and "Presidential

Medal of Freedom") are in correct parallel format.

(21) (C) This sentence corrects the original parallelism error but introduces a new error.

"The most famous leader of the American civil rights movement" is no longer placed in a list

with the two prizes won; instead, it is correctly used as a clause modifying "Martin Luther King Jr." However, "won the Nobel Peace Prize and he won the Presidential Medal of 

Freedom" is unnecessarily wordy; because the two are both prizes, they should be more

concisely presented in parallel format.

Page 25: GMAT Verbal Ques

7/30/2019 GMAT Verbal Ques

http://slidepdf.com/reader/full/gmat-verbal-ques 25/40

(22) (D) The items in a list require parallelism. The first two items ("the most famous

leader..." and "the winner of...") are parallel but the third item, "he won the Presidential Medal

of Freedom," is not.

(23) (E) "As well as the Presidential Medal of Freedom, too" is redundant.

(24) The original sentence contains two instances of redundant wording. First, we have

"typically causes death within 3 to 5 years of the onset of symptoms, on average..."

"Typically" and "on average" are both supplying the same information and are, therefore,

redundant. In addition, the word "although" at the beginning of the sentence indicates that a

contrast will appear later in the sentence. In the second half, we also have "in contrast,"

which supplies the same information as "although." Again, this is redundant.

(25) (A) This choice is incorrect as it repeats the original sentence.

(26) (B) This choice removes "typically," thereby eliminating the first redundancy problem

with "typically" and "on average." However, the sentence retains "in contrast." Since

"although" is not underlined and cannot be removed from the sentence, "in contrast" must be

removed in order to eliminate the redundancy.

(27) (C) CORRECT. This choice removes "on average," thereby eliminating the first

redundancy problem with "typically" and "on average." This choice also removes "in

contrast," thereby eliminating the second redundancy problem with "although and "in

contrast."

(28) (D) This choice removes "typically," thereby eliminating the first redundancy problem

with "typically" and "on average." The new placement of "average,"

however, incorrectly alters the meaning of the sentence, implying that sufferers will die within

3 to 5 years of the worldwide average onset, rather than within 3 to 5 years of the onset of 

their own disease. In addition, we still have the second redundancy problem ("although" and

"by contrast") and a new problem is created by the use of "by contrast." In this usage, "in

contrast" is the correct idiom.

(29) (E) This choice removes "in contrast," thereby eliminating the second redundancy

problem with "although and "in contrast." However, "typically" and "average" are both still

present in the sentence and are, therefore, still redundant. In addition, the new placement of 

"average" incorrectly alters the meaning of the sentence, implying that sufferers will die

within 3 to 5 years of the worldwide average onset, rather than within 3 to 5 years of the

onset of their own disease.

Page 26: GMAT Verbal Ques

7/30/2019 GMAT Verbal Ques

http://slidepdf.com/reader/full/gmat-verbal-ques 26/40

(30) In the second paragraph, the author mentions that frog, toad, and salamander populations

have declined, possibly due to environmental effects. The author then indicates that

amphibians have ―permeable skin [that] makes them unusually sensitive to environmental

changes‖ and suggests that amphibians are ―the biological harbingers of the natural world,

giving humans early notification‖ of environmental changes.‖ The word ―harbinger‖ means to

foreshadow or provide early notice of something that is going to happen.

(A) Permeable skin is introduced not to argue that other amphibians are endangered, but to

explain why amphibians in general are especially sensitive to changes to the environment.

(B) CORRECT. The author does suggest that amphibians’ special sensitivity—due at least in

part to their permeable skin—means that they are affected by adverse environments earlier 

than are other animals.

(C) The first paragraph does say that saving the golden toad’s habitat was not enough to

save the species, but this does not explain why the author discusses the permeability of 

amphibian skin in the second paragraph.

(D) The passage does suggest that permeable skin leaves amphibians especially

susceptible to their environment, but draws no distinction among various causes or types of 

adverse environments.

(E) The relevant sentence indicates that amphibians may give ―humans early notification of 

the deterioration of our ecosystem.‖ This does not suggest that humans can ignore threats to

amphibians; if anything, it suggests the opposite. (31) The first paragraph focuses predominately on the disappearance of a specific amphibian: the

golden toad of Costa Rica. The second paragraph explains that the demise of the golden

toad is one example of a global disappearance of amphibians, notes potential causes, andsuggests that the global phenomenon may be important to non-amphibian species.

(A) The first paragraph focuses on the disappearance of species in a particular area of the

world, while the second paragraph discusses potential causes and the significance of a

global phenomenon.

(B) No solutions that might mitigate further environmental damage are discussed in the

second paragraph of the passage.

(C) CORRECT. The first paragraph focuses primarily on Costa Rica’s golden toad, one

example of the widespread decline of amphibious populations in recent decades. Thesecond paragraph discusses possible causes for this decline as well as its possible

significance.

(D) The first paragraph does mention efforts to protect the habitat of the golden toad. In the

second paragraph, however, the passage does not advocate any action, or even discuss

such advocacy..

Page 27: GMAT Verbal Ques

7/30/2019 GMAT Verbal Ques

http://slidepdf.com/reader/full/gmat-verbal-ques 27/40

(E) The first paragraph focuses primarily on one particular toad in Costa Rica and never 

mentions the ―worldwide  disappearance‖ of toads. The second paragraph does mention that

salamanders and frogs are also disappearing, but this is not the focus of the paragraph. (32) ―Except‖ questions are negatively-worded questions. Typically, four of the answers will fall

into a certain category or be ―true‖ in some way, while the fifth choice will be the ―odd one

out.‖ The correct answer on EXCEPT questions will be the odd one out. Keep track of your eliminations on your scrap paper. Because it is often easier to find the four ―true‖ answer 

choices, focus on eliminating wrong answers rather than trying to find the one right answer.

(A) True. The second sentence of the second paragraph characterizes ―global warming‖ as a

side-effect of human activities.

(B) True. The first sentence of the second paragraph classifies both toads and frogs as

amphibians. Later in the second paragraph, the passage explains that amphibians have

permeable skin.

(C) True. The third sentence of the first paragraph states, ―setting aside habitat was notenough to save the species.‖ The golden toad in fact became extinct after M onteverde was

established.

(D) True. The second sentence of the first paragraph states that money was raised in the

United States to establish the Monteverde Cloud Forest Preserve.

(E) CORRECT. Unknown. The passage offers no information about the number of 

salamander species in Costa Rica that have disappeared since the late 1980s. We cannot

assume that this omission indicates a lesser number than either toad or frog species. (33) The phrase ―recent global boom‖ describes something that began in the past and is

continuing into the present; hence, the present perfect tense ―has led‖ is more appropriatethan the simple present tense ―leads.‖ In addition, ―such as‖ is preferred to ―like‖ when

introducing examples.

(A) This choice is incorrect as it repeats the original sentence.

(B) The singular subject ―boom‖ does not agree with the plural verb ―have led.‖ 

(C) The meaning of the sentence has changed because this choice states that the number of 

―thieves‖ has increased rather than the number of  ―thefts.‖ In addition, ―such as‖ is preferred

to ―like‖ when introducing examples. 

(D) CORRECT. This choice correctly uses the present perfect tense ―has led‖ to indicate that

the boom began in the past and is continuing into the present, and also to properly indicate

that the boom started before the rise in theft. In addition, ―such as‖ is preferred to ―like‖ when

introducing examples.

(E) The present perfect tense ―has led‖ is more appropriate than the present progressive

tense ―is leading‖ because although the pr esent progressive indicates an action or state that

Page 28: GMAT Verbal Ques

7/30/2019 GMAT Verbal Ques

http://slidepdf.com/reader/full/gmat-verbal-ques 28/40

is continuing, it does not properly indicate the relative time sequence of the boom and the

subsequent rise in the theft of metal objects. (34) (A) This choice is incorrect as it repeats the original sentence.

(B) The modifying phrase ―Discouraged by new data that show increases in toxic emissions

from domestic factories‖ is meant to modify the noun ―shareholders.‖ Therefore,―shareholders‖ should be placed directly after ―factories.‖ Instead, it seems that the

―searches‖ are ―Discouraged by new data,‖ which is not logical. Also, the passive

construction ―are being conducted by‖ is unnecessarily wordy. Finally, the placement of ―who

are looking for alternative investment opportunities‖ after ―companies‖ makes it seem that the

―companies‖ are ―searching for alternative investment opportunities.‖ According to the

original sentence, the ―shareholders‖ are looking for these ―opportunities,‖ not the

―companies.‖ 

(C) While the misplaced modifier issue is corrected by placing ―shareholders‖ adjacent to the

modifying phrase, the past perfect form of the verb, ―had begun,’’ is used unnecessarily. In

fact, the use of ―had begun‖ implies that the ―shareholders‖ had begun searching for newinvestment opportunities before the discouraging data were released. This is not the

intended meaning of the sentence. Also, ―investment opportunities outside of the

manufacturing industry‖ is wordy when compared with ―alternative investment opportunities.‖  

(D) The placement of ―the nation’s leading manufacturing companies‖ adjacent to the

modifying phrase makes it seem that these companies are ―Discouraged by new data,‖

which changes the meaning of the sentence. The original meaning is further compromised

by ―companies are searching.‖ The ―shareholders‖ are searching for new opportunities, not

the companies.

(E) CORRECT. The misplaced modifier issue is corrected by placing ―shareholders‖ adjacentto the modifying phrase. It is clear that the ―shareholders‖ are ―searching,‖ and not the

companies. The active voice "are searching" replaces the wordy passive construction

"searches. . . are being conducted by." Finally, the phrase ―alternative investment

opportunities‖ is clear and concise. (35) First, the subject of the sentence is "capital gains tax," which is singular. The verb,

however, is "have been being", which is plural. So we need to find a choice that contains a

singular verb. Second, we do not need "being" to indicate that the tax became the subject of 

debate at some point in the recent past and has continued to be the subject of debate up to

the present time.

(36)

(37) (A) This choice is the same as the original sentence.

(38)

(39) (B) "Have been" is plural.

(40)

(41) (C) "Were" is plural.

(42)

(43) (D) "Has been being" is singular, though "being" is unnecessary.

Page 29: GMAT Verbal Ques

7/30/2019 GMAT Verbal Ques

http://slidepdf.com/reader/full/gmat-verbal-ques 29/40

(44)

(45) (E) CORRECT. "Has been" is singular and in the proper tense.The question asks us to determine why the author mentions that nutmeg became a luxury item in the

European market. In the first paragraph, after establishing the ―luxury item‖ info, the author writes

―seeking a monopoly over this valuable spice, the Dutch attacked the Banda Islands.‖ 

(A) The passage does not discuss the trade of luxury items in general; it mentions only one luxury

item, nutmeg.

(B) CORRECT. The Dutch sought ―a monopoly‖ over the market for ―this valuable spice.‖ ―Seeking a

monopoly‖ is a way to indicate that the Dutch wanted to control the market for nutmeg.  

(C) The fact that nutmeg became a luxury item is the reason the Dutch wanted to monopolize the

market. It does not establish that the Dutch actuallysucceeded in establishing a monopoly.

(D) The author mentions only that nutmeg became a luxury item in the European market; there is no

mention made of other spices or their importance.

(E) There is no mention made in the passage of any desire on the part of the British to control the

nutmeg trade. No reason is given for their possession of one of the Banda Islands.

The correct answer to a ―main idea‖ question should reflect the overall message, or point, of the

passage. At the beginning of the first paragraph, the author makes the claim that "items that seem

unremarkable today might once have altered the course of history." In the next sentence, the author 

offers nutmeg as an example of that opening claim. The rest of the passage then elaborates on that

example.

(A) The passage is not primarily about the history of a major city, as it only mentions New

 Amsterdam/New York in passing.

(B) The passage provides only one example of the impact a commonplace item had on world

history; it does not discuss the role of multiple ―commonplace items.‖  

(C) CORRECT. Nutmeg is used as a specific example of the author's general claim that ―items that

seem unremarkable today might once have altered the course of history.‖ 

(D) The author does not argue for continued research into history, political or otherwise.

(E) While the main claim of the passage (that seemingly unremarkable items might have altered the

course of history) might reasonably be called innovative, this answer choice applies the ―innovative‖

label specifically to the nutmeg example. But the passage does not present an ―innovative view of‖

nutmeg (the commonplace item); the information about nutmeg is historical and fact-based in nature

The first paragraph says ―Seeking a monopoly over this valuable spice, the Dutch attacked the

Banda Islands.‖ The correct answer choice should paraphrase this in some way. 

(A) The passage does not mention increased competition with Britain as a reason for the initial

Page 30: GMAT Verbal Ques

7/30/2019 GMAT Verbal Ques

http://slidepdf.com/reader/full/gmat-verbal-ques 30/40

interest of the Netherlands in the Bandas; rather, increased competition was a result of the two

countries’ interest in the Bandass 

(B) The passage indicates that the Dutch subjugated the native people so that it could control the

nutmeg trade; the interest was in trade, not the subjugation of the native people for its own sake.

(C) The passage indicates that nutmeg became a luxury item in the European market; that is, the

Dutch were shipping nutmeg from the Banda Islands back to Europe. So though they were

harvesting nutmeg in a new geographical location, they did not (as far as we know) have a new

geographical market in which to sell their products.

(D) The passage does not mention any attempts to cultivate nutmeg outside of the Banda Islands.

(E) CORRECT. The Netherlands attacked the Bandas in order to control trade in nutmeg, a valuable

commodity at that time. It hoped to establish ―a monopoly over this valuable spice,‖ thus restricting

access to it.

The argument may be summarized as follows:

Exit poll: 4/5ths said they would vote for candidate who uses wheelchair 

Conclusion: poll may overestimate those who would vote for a candidate who uses a wheelchair 

It is helpful before proceeding to the answer choices to attempt to anticipate the missing portion of 

the argument: an unstated premise that would support our conclusion. Why would the poll have

overestimated those who would vote for a candidate who uses a wheelchair? Perhaps because

voters, wanting to look good for the pollster, were not truthful in answering the pollster’s questions. 

 A: This explains why the poll would have overestimated the proportion of voters who would vote for a

candidate who uses wheelchair. Correct.

B: This choice implies that people who would vote for a candidate who uses wheelchair told the

pollster they wouldn’t, which would cause the poll to undercount, not overcount, the proportion of 

voters who would vote for a candidate who uses wheelchair. Incorrect.

C: Providing additional information about people who reported that they would not vote for a

candidate who uses wheelchair does not support the conclusion that the proportion of people who

said they would vote for a candidate who uses a wheelchair was overestimated. Incorrect.

D: The conclusion is about registered voters; this statement is irrelevant. Incorrect.

E: The idea that voters who claimed that they would not vote for a candidate who uses wheelchair 

might change their minds under some circumstances supports the idea that the proportion of voters

Page 31: GMAT Verbal Ques

7/30/2019 GMAT Verbal Ques

http://slidepdf.com/reader/full/gmat-verbal-ques 31/40

who would vote for a candidate who uses wheelchair has been undercounted, not

overcounted. Incorrect.

The conclusion is that "one will have a wider selection of homes to choose from if one looks for a

home in Florida rather than in Texas." Why? Because 15% of all homes in Florida are on the market

whereas only 7% of all homes in Texas are on the market. This argument confuses percentages with

specific numbers. It is possible that 15% of the number of homes in Florida is actually smaller than

7% of the number of homes in Texas. If the number of homes in both states were the same, or if the

number of homes in Florida were greater than the number in Texas, the argument would be

stronger.

(A) The argument focuses on the number of homes, not their cost.

(B) Again, the argument focuses on the number of homes, not their affordability.

(C) If the construction industry in Texas is growing, the gap between the number of homes in Floridaand the number of homes in Texas might shrink, thus weakening the argument rather than

strengthening it.

(D) The argument does not focus on the cost of homes.

(E) CORRECT. This choice tells us that the absolute number of homes in Florida is greater than in

Texas. Therefore, 15% of the homes in Florida must represent a larger number of homes than does

7% of the homes in Texas, thus strengthening the argument.The symptoms of Kleine-Levin syndrome are presented in a list. The modified nouns ―extreme

lethargy‖ and ―heightened irritability‖ are parallel with each other, but ―appearing confused‖ must be

changed to ―apparent confusion‖ to make the third symptom parallel to the first two. 

(A) This choice is incorrect as it repeats the original sentence.

(B) The phrase ―they appear confused‖ is not parallel with the modified nouns ―extreme lethargy‖ and

―heightened irritability.‖ In addition, the plural pronoun ―they‖ has no clear antecedent; it could refer 

to either ―adolescents‖ or  ―episodes.‖ 

(C) The phrase ―may be confused‖ is not parallel with the modified nouns ―extreme lethargy‖ and

―heightened irritability.‖ In addition, the clause ―may be confused‖ has no clear subject, thus could

incorrectly suggest that the syndrome itself may be confused.

(D) CORRECT. The modified noun ―apparent confusion‖ is parallel with the other listed symptoms,

―extreme lethargy‖ and ―heightened irritability.‖ 

(E) The phrase ―had been confused‖ is not parallel with the modified nouns ―extreme lethargy‖ and

―heightened irritability.‖ In addition, the clause ―had been confused‖ has no clear subject, thus could

incorrectly suggest that the syndrome itself had been confused.

Page 32: GMAT Verbal Ques

7/30/2019 GMAT Verbal Ques

http://slidepdf.com/reader/full/gmat-verbal-ques 32/40

 A person's immune system becomes hypersensitive to an allergen via repeated exposure. We need

to find an example in which a series of minor events (repeated exposure to an allergen) can

ultimately provoke a typically unexpected, extreme response (a life-threatening reaction due to only

minimal exposure to the allergen).

(A) In this situation, a behavior (diet) over time leads one to become more susceptible to other chronic conditions (diabetes and high blood pressure). This does not mimic the idea of a series of 

minor events causing an unexpected and extreme reaction; rather, the process described is

reasonably to be expected.

(B) CORRECT. A series of minor earthquakes (minor events) weaken the existing infrastructure.

The weakened infrastructure then allows a minor earthquake to do much greater damage than

expected (unusual and extreme).

(C) This choice is tempting because it addresses an allergy health issue, but we want to mimic the

structure of the argument, not merely the topic. This choice says that someone with an allergy can

have a reaction by touching the substance to which he or she is allergic; this is to be expected andcannot reasonably be considered unexpected. Further, this choice does not discuss any sudden

onset of extreme susceptibility to the allergen.

(D) In this situation, a gradual occurrence (warming weather) leads to severe weather conditions.

We are not given information as to whether the increase in temperature is "minor," nor do we have

reason to know whether the response (severe weather) should be considered unexpected.

(E) It is reasonable to expect that the more someone speeds, the more likely she or he is to get

caught; this situation cannot reasonably be considered unexpected, nor is the result (getting caught)

extreme.

This argument provides that albinos, or people whose bodies do not produce melanin, are unusually

susceptible to solar exposure. This suggests a connection between the production of melanin in

humans and protection from sunburn and other sun-related ailments; we should look for a

conclusion that draws this connection.

(A) The argument does not indicate that people born with albinism somehow develop other natural

defenses against sun-related health issues.

(B) This conclusion is too extreme to be supported by the argument; nothing in the argument

suggests that humans whose bodies produce high levels of melanin can "easily ignore" sunburn or 

other sun-related health issues.

(C) There is no indication in the argument that sunburn reduces melanin production.

(D) CORRECT. The argument does strongly suggest that melanin plays some role in protecting the

skin from developing sunburn and other sun-related ailments, since albinos do not produce melanin

and are unusually susceptible to sun-related ailments.

Page 33: GMAT Verbal Ques

7/30/2019 GMAT Verbal Ques

http://slidepdf.com/reader/full/gmat-verbal-ques 33/40

(E) It is not suggested in the argument that an albino person could not protect him or herself from

solar exposure through artificial means, e.g. wearing protective clothing or powerful sunblock.

The correlative pair ―not only X but also Y‖ can be used to describe the two potential effects of 

regular vehicle maintenance: ―can not only extend the life…but also boost fuel efficiency.‖ These

effects should be structurally and logically parallel. The original sentence is incorrect because

―extend the life‖ is not structurally parallel to ―can boost fuel efficiency.‖ The second ―can‖ after ―but

also‖ is redundant because there is already a ―can‖ before ―not only,‖ causing the sentence to

illogically read ―regular maintenance can …can boost fuel efficiency.‖ 

(A) This choice is incorrect as it repeats the original sentence.

(B) CORRECT. This sentence uses the ―not only X but also Y‖ construction correctly and expresses

the two effects in logically and structurally parallel ways: ―extend the life‖ and ―boost fuel efficiency.‖ 

(C) ―But also‖ cannot be used alone; it must be used as part of the correlative pair ―not only X but

also Y.‖ Additionally, the second ―can‖ after ―but also‖ is redundant because there is already a ―can‖

before ―not only,‖ causing the sentence to illogically read ―regular maintenance can …can boost fuel

efficiency.‖ 

(D) ―Not only X but Y‖ is incomplete; ―also‖ is required in the correlative pair construction.

 Additionally, ―the life of a vehicle‖ is not structurally parallel to ―boost fuel efficiency.‖ Additionally, the

second ―can‖ after ―but‖ is redundant because there is already a ―can‖ before ―not only,‖ causing the

sentence to illogically read ―regular maintenance can …can boost fuel efficiency.‖ 

(E) ―But also‖ is required after ―not only,‖ but this sentence uses the incorrect construction ―not only

X and also Y.‖ Additionally, the second ―can‖ after ―and also‖ is redundant because there is already a

―can‖ before ―not only,‖ causing the sentence to illogically read ―regular maintenance can …can

boost fuel efficiency.‖ 

The legislators want to promote ethanol production in order to lessen our dependence on foreign oil.

 Any evidence that ethanol production would not lessen our dependence on foreign oil would

undermine the legislators' conclusion.

(A) CORRECT. If it takes 1.5 gallons of oil to make 1 gallon of ethanol, it is not clear that producing

ethanol can help us to reduce our reliance on foreign oil.

(B) As long as many drivers prefer not to switch to electric cars, it is possible that the ethanol fuel

mix will come into widespread use.

(C) Although it may be expensive to retrofit a non-ethanol vehicle, ethanol-ready new vehicles might

constitute a large potential market for subsidized ethanol.

(D) This answer choice is irrelevant because the argument does not mention pollution.

(E) Without any information as to why the ethanol/gasoline blend has not been widely adopted in

Europe, we cannot reach any conclusions that are relevant to the United States.

Page 34: GMAT Verbal Ques

7/30/2019 GMAT Verbal Ques

http://slidepdf.com/reader/full/gmat-verbal-ques 34/40

This sentence has three mistakes. First, the idiom ―not only X but also Y‖ is not properly used here.

Second, the two parts compared should be parallel to each other; i.e., ―finding good schools‖ and

―knowing what makes …." should be used. Finally, the pronoun ―one‖ does not have a clear 

antecedent: it could refer to either schools or children.

(A) This choice is incorrect as it repeats the original sentence.

(B) This choice does not correct the improperly used idiom ―not only X but also Y.‖ In addition, it

does not use parallel structure for ―X‖ ("finding") and ―Y" ("must decide").  

(C) This choice does not correct the improperly used idiom ―not only X but also Y.‖ It also changes

the meaning of the sentence by using the phrase ―decide which school is best of them all.‖ 

(D) This choice does not correct the improperly used idiom ―not only X but also Y.‖ In addition, it

does not use parallel structure for ―X‖ ("finding") and ―Y" ("have to know"). 

(E) CORRECT. This choice corrects all three errors in the original sentence. First, it uses the idiom―not only X but also Y‖ correctly. Second, it uses parallel structure in the phrase ―not only with finding

… but also with knowing …. ‖ Finally, the phrase ―one school‖ makes it clear that ―one‖ refers to

―school.‖ 

On questions asking about the main idea of the passage, it is useful to refer to the opening

paragraph, which usually provides a broad overview of the passage. Also, be sure to avoid extreme

answer choices and those answers that refer to a part of the passage rather than the whole text.

(A) While the passage discusses the origin of the women’s rights movement, the text does not

attempt to compare it to the contemporary state of affairs. Nothing in the passage is mentioned

about the current-day situation.

(B) While the passage discusses the development of the women’s rights movement, it does not

suggest further expansion of women’s rights. 

(C) The passage mentions only a few restrictions on women’s rights and does so in a cursory way.

Furthermore, the text merely describes rather than criticizes these restrictions.

(D) The passage provides no information regarding the specific reasons for the opposition to

women’s rights movement. 

(E) CORRECT. The entire passage is devoted to the discussion of the early days in the women’s

rights movement and the events leading up to the Seneca Falls Convention, which, according to the

passage, "is commonly regarded as the beginning of the women's rights movement in the United

States." Note that this idea is mimicked in the first sentence of the opening paragraph: ―The

movement for women’s rights traces its origin to the first half of the nineteenth century.‖ 

On questions that ask about the purpose of including a certain piece of information in the passage,

make sure to evaluate the context in which this evidence is presented. Therefore, it is useful to re-

read not only the sentence that provides the information at issue, but also one or two sentences

Page 35: GMAT Verbal Ques

7/30/2019 GMAT Verbal Ques

http://slidepdf.com/reader/full/gmat-verbal-ques 35/40

before and after it.

(A) The passage provides no information about the nature of the personal relationship between

Stanton and her husband.

(B) Nothing in the passage suggests that Stanton's husband needed to leave town to attend another convention.

(C) While such a radical statement by Stanton’s husband may reasonably suggest that he was prone

to emotional and spontaneous decisions, it is not the reason for mentioning this information. As

implied by the second sentence of the third paragraph, the author mentions this information to

emphasize the boldness of content in the Declaration of Sentiments.

(D) CORRECT. The second sentence of the third paragraph states that ―… at the time of its

composition, the Declaration of Sentiments was so bold that … he stated that if she read it at the

Seneca Falls Convention, he would have to leave town.‖ Thus, the author mentions the intention of 

Stanton’s husband to leave town in order to support the earlier statement about the bold content of the Declaration of Sentiments.

(E) Nothing in the passage suggests that the Declaration of Sentiments contained abusive language.

When answering all reading comprehension questions, be sure to avoid any words that are not

 justified by the passage.

To answer this detail question, use the signal word ―Declaration of Sentiments‖ to locate the

appropriate sentence in the third paragraph. Specifically, the penultimate sentence in the third

paragraph states that the Declaration of Sentiments ―proclaimed that all men and women are born

equal and stated that that no man could withhold a woman's rights, take her property, or preclude

her from the right to vote.‖ Since this is an ―except‖ question, we can go through the answer choices

and eliminate those statements that are mentioned in the support sentence, leaving us with the

correct answer.

(A) CORRECT. Based on the penultimate sentence of the third paragraph, no information is

provided about the Declaration’s treatment of women’s rights to participate in military expeditions. 

(B) According to the penultimate sentence of the third paragraph, the Declaration of Sentiments

proclaimed that ―… no man could … preclude her [a woman] from the right to vote.‖  

(C) According to the penultimate sentence of the third paragraph, the Declaration of Sentiments

proclaimed that ―… no man could … take her [woman’s] property...‖ 

(D) According to the penultimate sentence of the third paragraph, the Declaration of Sentiments

proclaimed that ―… all men and women are born equal ...‖ 

(E) According to the penultimate sentence of the third paragraph, the Declaration of Sentiments

proclaimed that ―... no man could withhold a woman's rights 

Page 36: GMAT Verbal Ques

7/30/2019 GMAT Verbal Ques

http://slidepdf.com/reader/full/gmat-verbal-ques 36/40

The conclusion of the passage is that the number of hybrid cars sold annually is likely to flatten over 

the next few years. This conclusion is supported by the premise that Congress has decreased tax

incentives for buyers of hybrid cars. The necessary assumption linking this premise to the conclusion

is that buyers are motivated to buy hybrid cars primarily because they can save money in the long

run. The answer choice that undermines this assumption will most weaken the conclusion of the

passage.

(A) CORRECT. This answer choice weakens the conclusion by undermining the assumption upon

which it is based.

(B) This answer choice provides an additional reason for buyers motivated by cost savings not to

purchase hybrid cars, and therefore helps to strengthen the conclusion rather than weaken it.

(C) One of the attractions of hybrid cars is their fuel efficiency. As the price of gasoline increases, so

does the cost-savings per mile driven of hybrid cars compared to traditional gasoline-powered cars.

The fact that gasoline prices are forecast to fall over the next few years means that hybrid cars offer 

less of a benefit than they would in an environment with rising gasoline prices. Hence, this doesmore to strengthen the conclusion rather than to weaken it.

(D) This choice is irrelevant to the tax-based argument about hybrid sales. However, it does give an

additional reason for buyers to avoid purchasing a hybrid car and, if anything, slightly supports the

prediction that hybrid sales will flatten.

(E) This choice is irrelevant to the tax-based argument about hybrid sales. In addition, perhaps the

costs of this advertising campaign will be passed on to consumers, making hybrid cars even more

expensive.

The modifying phrase ―hoping to alleviate some of the financial burdens…‖ begins this sentence and

should be followed immediately by the noun the modifier refers to, ―the county government.‖

However, the original sentence illogically suggests that ―property taxes‖ were hoping to alleviate the

financial burdens. Additionally, the phrase ―raised by an eleven percent increase‖ contains a

redundancy; either ―raised by eleven percent‖ or ―increased by eleven percent‖ would be more

concise and correct. Finally, the passive construction ―property taxes…were raised…by the county

government‖ is wordier than the preferred active construction ―the county

government…raised…property taxes.‖ 

(A) This choice is incorrect as it repeats the original sentence.

(B) The modifying phrase ―hoping to alleviate…‖ should be followed immediately by the noun the

modifier refers to, ―the county government.‖ However, this choice illogically suggests that ―property

taxes‖ were hoping to alleviate the financial burdens. Also, the passive construction ―property

taxes…were raised…by the county government‖ is wordier than the preferred active construction

―the county government…raised…property taxes.‖ 

(C) The phrase ―raised…by an eleven percent increase‖ contains a redundancy; either ―raised by

eleven percent‖ or ―increased by eleven percent‖ would be more concise and correct. 

Page 37: GMAT Verbal Ques

7/30/2019 GMAT Verbal Ques

http://slidepdf.com/reader/full/gmat-verbal-ques 37/40

(D) The phrase ―last year raised by eleven percent property taxes‖ is awkward, since ―property

taxes,‖ the object, do not immediately follow the verb ―raised.‖ The meaning would be clearer if it

were phrased ―raised property taxes by eleven percent last year.‖ 

(E) CORRECT. This choice is the most concise and correct. ―The county government‖ correctly

follows the modifying phrase ―hoping to alleviate…‖ The concise phrase ―raised…by eleven percent‖is used. Finally, the active construction ―the county government…raised…property taxes‖ replaces

the wordier passive construction ―property taxes…were raised…by the county government.‖ 

First, the subject of the main clause is "the increase in unemployment rates," which is singular. Yet,

the verb in the main clause is "are forcing," which is plural. Second, "significantly increased retail

prices as well as energy costs" is both awkward and unclear. The logical inference is that the energy

costs have increased as well, but this is not clear from the grammar that "significantly increased"

modifies "energy costs" in addition to "retail prices." This phrase could also be more elegantly

expressed.

(A) This choice is incorrect as it repeats the original sentence.

(B) The subject of the main clause here is "the increase in rates of unemployment," which is singular,

but the verb is "have been forcing," which is plural. However, the replacement of "as well as" with

"and" is an improvement over the original sentence because it more clearly links "significant

increases" and "energy costs."

(C) CORRECT. The subject of the main clause is "the increase in unemployment rates" and the

corresponding verb is "is forcing," which are both singular. The phrase "coupled with significant

increases in both retail prices and energy costs" is elegant and clearly links "significant increases" to

"energy costs" through the use of "both."

(D) The subject of the main clause is "the increase in unemployment rates" and the corresponding

verb is "is forcing," which are both singular. However, the phrase "coupled with significantly

increased retail prices as well as energy costs" is still awkward and unclear.

(E) The subject of the main clause is "the increase in unemployment rates" and the corresponding

verb is "had been forcing," which are both singular. However, "had been forcing" is in the past

perfect tense, which requires two past actions, one of which must occur earlier than than the other.

This is not the case here. Moreover, the correct idiom is "both X and Y" and not "both X as well as

Y."

We are given a list of problematic events, each one leading to the next, and we are asked to find a

way to break the cycle of events. In order to do so, we need to find a way to alter one of these

events in such a way that it will change the overall cycle so that small community hospitals are no

longer losing money (at least, for these particular reasons).

Page 38: GMAT Verbal Ques

7/30/2019 GMAT Verbal Ques

http://slidepdf.com/reader/full/gmat-verbal-ques 38/40

 

(A) The reimbursement rates for specialty care are not a part of the problematic cycle; rather, the

relatively low volume of specialty care at community hospitals is the problem. If the hospital is doing

very little or no specialty care work in the first place, then charging more for specialty care won't help

much.

(B) Even if the hospital successfully advertises its specialty care services and attracts a lot of new

customers, it will not be able to serve these customers because it does not have the necessary

specialists in place to provide the specialty care. This choice does not "follow through" the entire

cycle.

(C) CORRECT. Since the local residents choose to patronize the nearby affluent hospitals due to the

presence of specialists, they will also begin to patronize the community hospital, which is using

these same specialists. The amount of high-margin specialty care will increase and the hospital will

make more money, thus enabling it to continue using specialists.

(D) The mere act of educating the public about the relatively low reimbursement rates of governmenthealth plans does not directly impact the hospital or break the cycle. In order for this to work, the

hospital would have to find some way to increase reimbursement rates for routine care from the

government plans or convert people from government plans to private insurers that reimburse at

better rates.

(E) Most of the community hospital's patients are either on government-funded plans or lack

insurance altogether. Receiving more money from private insurers, therefore, is unlikely to generate

enough revenue to break the cycle.

The argument provides seemingly conflicting evidence. On the one hand, the class size remained

constant. On the other hand, the college received a greater number of applications and a greater 

proportion of applicants received offers of admission from the college. We need to select an answer 

that would reconcile this contradiction.

(A) While this statement explains the increase in applications to College X, it provides no information

about why the size of the entering class remained the same despite an increase in both the number 

of applications and the acceptance rate.

(B) While this answer choice explains one of the reasons for the increase in the applications to

College X, it provides no information about why a greater proportion of applicants received an offer 

from the college.

(C) CORRECT. This statement explains how the size of the entering class at College X could remain

the same despite the rise in applications and increased acceptance rate. Since a substantially

greater proportion of accepted applicants decided to attend other colleges, College X had to extend

more offers to fill the same class. As a result, the admission rate increased despite the increase in

applications and no changes in the class size.

(D) The relative comparison of College X to other academic institutions is beyond the scope of the

argument.

Page 39: GMAT Verbal Ques

7/30/2019 GMAT Verbal Ques

http://slidepdf.com/reader/full/gmat-verbal-ques 39/40

 

(E) While this statement may explain the popularity of the college in general, it provides no

information justifying the increase in applications. Further, it does not explain why a greater prop

ortion of applicants received an offer of admission.

The original sentence is a run-on sentence, since it separates two independent clauses, each of 

which could be an independent sentence, with only a comma.

(A) This choice is incorrect as it repeats the original sentence.

(B) This choice separates the two clauses with a semicolon, which would be an appropriate choice

of punctuation if used between two independent clauses. In this choice, the clause after the

semicolon is not an independent clause; instead, it is a sentence fragment.

(C) CORRECT. This choice properly uses a semicolon to separate two independent clauses.

(D) On the GMAT, ―where‖ must refer to a specific location. Additionally, the plural pronoun ―they‖

has no clear plural antecedent.

(E) In this choice, the plural pronoun ―they‖ has no clear plural antecedent.

The director of programming argues that the addition of a conservative news program in response to

the hiring of a liberal news personality represents an institutional attempt at balancing different

perspectives, rather than any conservative bias. The evidence provided by the director is outside of 

the context of any past actions on the part of the Network; it would be useful to have more

information about actions the Network has taken in response to the hiring of conservative or liberal

news personalities in the past to determine a pattern of behavior.

(A) CORRECT. If the Network responds to the addition of a liberal news personality in the same

way that it does the addition of a conservative news personality, then the argument presented by the

director is valid. An identical response to the hiring of a personality from either side of the political

spectrum suggests that the Network does act in a fair and balanced manner, at least in this regard.

(B) Whether the Network presents a program airing the opinions of its viewers on contemporary

issues does not indicate either a balanced approach or a conservative bias. This statement is

irrelevant.

(C) The nature of the former employer of the newly-hired liberal news personality is not relevant in

determining whether or not there is a systematic conservative bias regarding the overall

programming of the Network.

Page 40: GMAT Verbal Ques

7/30/2019 GMAT Verbal Ques

http://slidepdf.com/reader/full/gmat-verbal-ques 40/40

(D) Whether the new liberal news personality has indicated that he has felt any editorial pressure is

not relevant; he might not want to admit to such pressure even if it did exist. Moreover, the

experience of one employee does not address the overall programming of the Network.

(E) That the Network has suffered from lower ratings in the past year is not relevant to determining

the bias or lack thereof of the Network, though it may explain why new personalities and

programming are being added.

The sentence compares "the portion of the interest" set aside to fund x to "that" (the portion of 

the interest) set aside to fund y . Since the "portion of the interest" is a singular quantity, it cannot be

described using the phrase "more numerous," which can be used only for plural nouns. (A

countable plural noun such as "students" could be described as "more numerous." For example, one

could say "The merit-scholarship students are more numerous than the athletic-scholarship

students.") To compare a singular quantity, the phrase "greater than" should be used instead.

(A) This choice is incorrect as it repeats the original sentence.

(B) This choice incorrectly uses the modifier "more numerous" to describe the singular quantity "the

portion of interest." Moreover, the plural verb "are" does not agree with the singular subject "the

portion of interest."

(C) CORRECT. This choice correctly uses the phrase "greater than" to compare two singular 

quantities: "The portion of interest" set aside to fund x "is greater than that" set aside to fund y .

(D) While this choice correctly uses the word "greater" to compare two singular quantities, it

incorrectly omits the word "than" thereby creating an incomplete comparison: "The portion of interest

. . . set aside to fund [x ] is greater that set aide to fund [y ]."

(E) This choice correctly uses the phrase "greater than" to compare two singular quantities: "The

portion of interest" set aside to fund x "is greater than that" set aside to fund y . However, the plural

verb "are" does not agree with the singular subject "the portion of interest."